返回列表 发帖

求一道逻辑题!~

Despite improvements in treatment for asthma, the death rate from this disease has doubled during the past decade from its previous rate. Two possible explanations for this increase have been offered. First, the recording of deaths due to asthma has become more widespread and accurate in the past decade than it had been previously. Second, there has been an increase in urban pollution. However, since the rate of deaths due to asthma has increased dramatically even in cities with long-standing, comprehensive medical records and with little or no urban pollution, one must instead conclude that the cause of increased deaths is the use of bronchial inhalers by asthma sufferers to relieve their symptoms.


Which one of the following is an assumption on which the argument depends?
(A) Urban pollution has not doubled in the past decade.
(B) Doctors and patients generally ignore the role of allergies in asthma.
(C) Bronchial inhalers are unsafe, even when used according to the recommended instructions.
(D) The use of bronchial inhalers aggravates other diseases that frequently occur among asthma sufferers and that often lead to fatal outcomes even when the asthma itself does not.
(E) Increased urban pollution, improved recording of asthma deaths, and the used of asthma deaths, and the use of bronchial inhalers are the only possible explanations of the increased death rate due to asthma.



选e..why not a?
谢谢!
收藏 分享

题目说两个原因中必须有一个被替换,假设既是此三个原因任意单独的一个都不能作为原因。
对c取非,即使药品b是安全的,但是药品b只是止疼药,患者仍可能因药品b死于疾病a,取非对原文不构成削弱。选项c因属于无关选项

TOP

I carefully noticed the E choice.
There are 4 choices in E.
Increased urban pollution,
improved recording of asthma deaths,
and the used of asthma deaths,
and the use of bronchial inhalers

Therefore, I exclude E because even if we exclude the first 2 reasons, we still have 2 . One is the used of astham deaths (I cannot understand this phrase, however.), and the other is the use of bronchinal inhalers.
Could you help me to make it clear? I exclude E, then turning to Choice C .

TOP

If bronchial inhaler is safe, the cause of increased deaths can still be the use of bronchial inhalers.

TOP

Weaken is not enough.  You have to make the conclusion WRONG.

TOP

Why?When C is neglected,I think it will weaken the conclusion.

TOP

A has no impact on the conclusion.

TOP

E is the answer.   There are only three choices for asthma death,

two of which were eliminated. The only one left standing is the use

of bronchial inhalers.

C is wrong because it is not an assumption for the conclusion to

hold. If you negate it, i.e., bronchial inhalers is safe to use, the

conclusion that the use of bronchial inhalers causes asthma death

still holds. So answer choice C has no impact on the conclusion of

the argument.

TOP

返回列表

站长推荐 关闭


美国top10 MBA VIP申请服务

自2003年开始提供 MBA 申请服务以来,保持着90% 以上的成功率,其中Top10 MBA服务成功率更是高达95%


查看